Is $mathbb R^J$ normal in the box topology when $J$ is uncountable?“Proof” that $mathbbR^J$ is not normal when $J$ is uncountableHow to prove that $mathbb R^omega$ with the box topology is completely regularcontinuity box topologyConvergence in the Box Topology.Normal spaces in box and uniform topologyWhat are the components and path components of $mathbbR^omega$ in the product, uniform, and box topologies?$mathbb R^omega$ in the box topology is not first countable.Uncountability of $mathbbR^I$ if $I$ is uncountableconnected components in box topologyMetrizability of infinite product space under box topologyIs $mathbbR^omega$ endowed with the box topology completely normal (or hereditarily normal)?

What is it called when someone votes for an option that's not their first choice?

Is VPN a layer 3 concept?

What are the rules for concealing thieves' tools (or items in general)?

Nested Dynamic SOQL Query

Which partition to make active?

How do researchers send unsolicited emails asking for feedback on their works?

How to test the sharpness of a knife?

What kind of footwear is suitable for walking in micro gravity environment?

How can an organ that provides biological immortality be unable to regenerate?

Do people actually use the word "kaputt" in conversation?

is this saw blade faulty?

Did Nintendo change its mind about 68000 SNES?

What are the consequences of changing the number of hours in a day?

How to balance a monster modification (zombie)?

Does fire aspect on a sword, destroy mob drops?

Have any astronauts/cosmonauts died in space?

UK Tourist Visa- Enquiry

Why are there no stars visible in cislunar space?

When did hardware antialiasing start being available?

Single word to change groups

What will the Frenchman say?

Why is indicated airspeed rather than ground speed used during the takeoff roll?

Is xar preinstalled on macOS?

Unfrosted light bulb



Is $mathbb R^J$ normal in the box topology when $J$ is uncountable?


“Proof” that $mathbbR^J$ is not normal when $J$ is uncountableHow to prove that $mathbb R^omega$ with the box topology is completely regularcontinuity box topologyConvergence in the Box Topology.Normal spaces in box and uniform topologyWhat are the components and path components of $mathbbR^omega$ in the product, uniform, and box topologies?$mathbb R^omega$ in the box topology is not first countable.Uncountability of $mathbbR^I$ if $I$ is uncountableconnected components in box topologyMetrizability of infinite product space under box topologyIs $mathbbR^omega$ endowed with the box topology completely normal (or hereditarily normal)?













4












$begingroup$


Question:




Is $mathbb R^J$ normal in the box topology when $J$ is uncountable?




I know $mathbb R^J$ is not normal in the product topology, see "Proof" that $mathbbR^J$ is not normal when $J$ is uncountable ;



I also know $mathbb R^omega$ is normal in the box topology assuming the continuum hypothesis, see Is it still an open problem whether $mathbb R^omega$ is normal in the box topology?.



That's the motivation for this problem. Unfortunately, the above two theorems don't imply anything about the normality of $mathbb R^J$. Any hint would be appreciated.










share|cite|improve this question











$endgroup$







  • 1




    $begingroup$
    What do you mean by "Box topology" ? It's not a standard naming...
    $endgroup$
    – Jean Marie
    Mar 13 at 12:12






  • 1




    $begingroup$
    @JeanMarie Maybe box product sounds better?
    $endgroup$
    – YuiTo Cheng
    Mar 13 at 12:13











  • $begingroup$
    A closed subspace of a normal space is normal. $Bbb R^omega$ is homeomorphic to a closed subspace of $Bbb R^k$ if $k$ is uncountable.
    $endgroup$
    – DanielWainfleet
    Mar 13 at 14:49










  • $begingroup$
    @DanielWainfleet So you are suggesting this problem is likely to be open, right?
    $endgroup$
    – YuiTo Cheng
    Mar 13 at 14:53










  • $begingroup$
    If the countable product isn't normal (which is open) then so would the higher powers be.
    $endgroup$
    – Henno Brandsma
    Mar 13 at 16:49















4












$begingroup$


Question:




Is $mathbb R^J$ normal in the box topology when $J$ is uncountable?




I know $mathbb R^J$ is not normal in the product topology, see "Proof" that $mathbbR^J$ is not normal when $J$ is uncountable ;



I also know $mathbb R^omega$ is normal in the box topology assuming the continuum hypothesis, see Is it still an open problem whether $mathbb R^omega$ is normal in the box topology?.



That's the motivation for this problem. Unfortunately, the above two theorems don't imply anything about the normality of $mathbb R^J$. Any hint would be appreciated.










share|cite|improve this question











$endgroup$







  • 1




    $begingroup$
    What do you mean by "Box topology" ? It's not a standard naming...
    $endgroup$
    – Jean Marie
    Mar 13 at 12:12






  • 1




    $begingroup$
    @JeanMarie Maybe box product sounds better?
    $endgroup$
    – YuiTo Cheng
    Mar 13 at 12:13











  • $begingroup$
    A closed subspace of a normal space is normal. $Bbb R^omega$ is homeomorphic to a closed subspace of $Bbb R^k$ if $k$ is uncountable.
    $endgroup$
    – DanielWainfleet
    Mar 13 at 14:49










  • $begingroup$
    @DanielWainfleet So you are suggesting this problem is likely to be open, right?
    $endgroup$
    – YuiTo Cheng
    Mar 13 at 14:53










  • $begingroup$
    If the countable product isn't normal (which is open) then so would the higher powers be.
    $endgroup$
    – Henno Brandsma
    Mar 13 at 16:49













4












4








4





$begingroup$


Question:




Is $mathbb R^J$ normal in the box topology when $J$ is uncountable?




I know $mathbb R^J$ is not normal in the product topology, see "Proof" that $mathbbR^J$ is not normal when $J$ is uncountable ;



I also know $mathbb R^omega$ is normal in the box topology assuming the continuum hypothesis, see Is it still an open problem whether $mathbb R^omega$ is normal in the box topology?.



That's the motivation for this problem. Unfortunately, the above two theorems don't imply anything about the normality of $mathbb R^J$. Any hint would be appreciated.










share|cite|improve this question











$endgroup$




Question:




Is $mathbb R^J$ normal in the box topology when $J$ is uncountable?




I know $mathbb R^J$ is not normal in the product topology, see "Proof" that $mathbbR^J$ is not normal when $J$ is uncountable ;



I also know $mathbb R^omega$ is normal in the box topology assuming the continuum hypothesis, see Is it still an open problem whether $mathbb R^omega$ is normal in the box topology?.



That's the motivation for this problem. Unfortunately, the above two theorems don't imply anything about the normality of $mathbb R^J$. Any hint would be appreciated.







general-topology separation-axioms box-topology






share|cite|improve this question















share|cite|improve this question













share|cite|improve this question




share|cite|improve this question








edited Mar 13 at 11:20







YuiTo Cheng

















asked Mar 13 at 9:54









YuiTo ChengYuiTo Cheng

2,0592637




2,0592637







  • 1




    $begingroup$
    What do you mean by "Box topology" ? It's not a standard naming...
    $endgroup$
    – Jean Marie
    Mar 13 at 12:12






  • 1




    $begingroup$
    @JeanMarie Maybe box product sounds better?
    $endgroup$
    – YuiTo Cheng
    Mar 13 at 12:13











  • $begingroup$
    A closed subspace of a normal space is normal. $Bbb R^omega$ is homeomorphic to a closed subspace of $Bbb R^k$ if $k$ is uncountable.
    $endgroup$
    – DanielWainfleet
    Mar 13 at 14:49










  • $begingroup$
    @DanielWainfleet So you are suggesting this problem is likely to be open, right?
    $endgroup$
    – YuiTo Cheng
    Mar 13 at 14:53










  • $begingroup$
    If the countable product isn't normal (which is open) then so would the higher powers be.
    $endgroup$
    – Henno Brandsma
    Mar 13 at 16:49












  • 1




    $begingroup$
    What do you mean by "Box topology" ? It's not a standard naming...
    $endgroup$
    – Jean Marie
    Mar 13 at 12:12






  • 1




    $begingroup$
    @JeanMarie Maybe box product sounds better?
    $endgroup$
    – YuiTo Cheng
    Mar 13 at 12:13











  • $begingroup$
    A closed subspace of a normal space is normal. $Bbb R^omega$ is homeomorphic to a closed subspace of $Bbb R^k$ if $k$ is uncountable.
    $endgroup$
    – DanielWainfleet
    Mar 13 at 14:49










  • $begingroup$
    @DanielWainfleet So you are suggesting this problem is likely to be open, right?
    $endgroup$
    – YuiTo Cheng
    Mar 13 at 14:53










  • $begingroup$
    If the countable product isn't normal (which is open) then so would the higher powers be.
    $endgroup$
    – Henno Brandsma
    Mar 13 at 16:49







1




1




$begingroup$
What do you mean by "Box topology" ? It's not a standard naming...
$endgroup$
– Jean Marie
Mar 13 at 12:12




$begingroup$
What do you mean by "Box topology" ? It's not a standard naming...
$endgroup$
– Jean Marie
Mar 13 at 12:12




1




1




$begingroup$
@JeanMarie Maybe box product sounds better?
$endgroup$
– YuiTo Cheng
Mar 13 at 12:13





$begingroup$
@JeanMarie Maybe box product sounds better?
$endgroup$
– YuiTo Cheng
Mar 13 at 12:13













$begingroup$
A closed subspace of a normal space is normal. $Bbb R^omega$ is homeomorphic to a closed subspace of $Bbb R^k$ if $k$ is uncountable.
$endgroup$
– DanielWainfleet
Mar 13 at 14:49




$begingroup$
A closed subspace of a normal space is normal. $Bbb R^omega$ is homeomorphic to a closed subspace of $Bbb R^k$ if $k$ is uncountable.
$endgroup$
– DanielWainfleet
Mar 13 at 14:49












$begingroup$
@DanielWainfleet So you are suggesting this problem is likely to be open, right?
$endgroup$
– YuiTo Cheng
Mar 13 at 14:53




$begingroup$
@DanielWainfleet So you are suggesting this problem is likely to be open, right?
$endgroup$
– YuiTo Cheng
Mar 13 at 14:53












$begingroup$
If the countable product isn't normal (which is open) then so would the higher powers be.
$endgroup$
– Henno Brandsma
Mar 13 at 16:49




$begingroup$
If the countable product isn't normal (which is open) then so would the higher powers be.
$endgroup$
– Henno Brandsma
Mar 13 at 16:49










0






active

oldest

votes











Your Answer





StackExchange.ifUsing("editor", function ()
return StackExchange.using("mathjaxEditing", function ()
StackExchange.MarkdownEditor.creationCallbacks.add(function (editor, postfix)
StackExchange.mathjaxEditing.prepareWmdForMathJax(editor, postfix, [["$", "$"], ["\\(","\\)"]]);
);
);
, "mathjax-editing");

StackExchange.ready(function()
var channelOptions =
tags: "".split(" "),
id: "69"
;
initTagRenderer("".split(" "), "".split(" "), channelOptions);

StackExchange.using("externalEditor", function()
// Have to fire editor after snippets, if snippets enabled
if (StackExchange.settings.snippets.snippetsEnabled)
StackExchange.using("snippets", function()
createEditor();
);

else
createEditor();

);

function createEditor()
StackExchange.prepareEditor(
heartbeatType: 'answer',
autoActivateHeartbeat: false,
convertImagesToLinks: true,
noModals: true,
showLowRepImageUploadWarning: true,
reputationToPostImages: 10,
bindNavPrevention: true,
postfix: "",
imageUploader:
brandingHtml: "Powered by u003ca class="icon-imgur-white" href="https://imgur.com/"u003eu003c/au003e",
contentPolicyHtml: "User contributions licensed under u003ca href="https://creativecommons.org/licenses/by-sa/3.0/"u003ecc by-sa 3.0 with attribution requiredu003c/au003e u003ca href="https://stackoverflow.com/legal/content-policy"u003e(content policy)u003c/au003e",
allowUrls: true
,
noCode: true, onDemand: true,
discardSelector: ".discard-answer"
,immediatelyShowMarkdownHelp:true
);



);













draft saved

draft discarded


















StackExchange.ready(
function ()
StackExchange.openid.initPostLogin('.new-post-login', 'https%3a%2f%2fmath.stackexchange.com%2fquestions%2f3146344%2fis-mathbb-rj-normal-in-the-box-topology-when-j-is-uncountable%23new-answer', 'question_page');

);

Post as a guest















Required, but never shown

























0






active

oldest

votes








0






active

oldest

votes









active

oldest

votes






active

oldest

votes















draft saved

draft discarded
















































Thanks for contributing an answer to Mathematics Stack Exchange!


  • Please be sure to answer the question. Provide details and share your research!

But avoid


  • Asking for help, clarification, or responding to other answers.

  • Making statements based on opinion; back them up with references or personal experience.

Use MathJax to format equations. MathJax reference.


To learn more, see our tips on writing great answers.




draft saved


draft discarded














StackExchange.ready(
function ()
StackExchange.openid.initPostLogin('.new-post-login', 'https%3a%2f%2fmath.stackexchange.com%2fquestions%2f3146344%2fis-mathbb-rj-normal-in-the-box-topology-when-j-is-uncountable%23new-answer', 'question_page');

);

Post as a guest















Required, but never shown





















































Required, but never shown














Required, but never shown












Required, but never shown







Required, but never shown

































Required, but never shown














Required, but never shown












Required, but never shown







Required, but never shown







Popular posts from this blog

Lowndes Grove History Architecture References Navigation menu32°48′6″N 79°57′58″W / 32.80167°N 79.96611°W / 32.80167; -79.9661132°48′6″N 79°57′58″W / 32.80167°N 79.96611°W / 32.80167; -79.9661178002500"National Register Information System"Historic houses of South Carolina"Lowndes Grove""+32° 48' 6.00", −79° 57' 58.00""Lowndes Grove, Charleston County (260 St. Margaret St., Charleston)""Lowndes Grove"The Charleston ExpositionIt Happened in South Carolina"Lowndes Grove (House), Saint Margaret Street & Sixth Avenue, Charleston, Charleston County, SC(Photographs)"Plantations of the Carolina Low Countrye

random experiment with two different functions on unit interval Announcing the arrival of Valued Associate #679: Cesar Manara Planned maintenance scheduled April 23, 2019 at 00:00UTC (8:00pm US/Eastern)Random variable and probability space notionsRandom Walk with EdgesFinding functions where the increase over a random interval is Poisson distributedNumber of days until dayCan an observed event in fact be of zero probability?Unit random processmodels of coins and uniform distributionHow to get the number of successes given $n$ trials , probability $P$ and a random variable $X$Absorbing Markov chain in a computer. Is “almost every” turned into always convergence in computer executions?Stopped random walk is not uniformly integrable

How should I support this large drywall patch? Planned maintenance scheduled April 23, 2019 at 00:00UTC (8:00pm US/Eastern) Announcing the arrival of Valued Associate #679: Cesar Manara Unicorn Meta Zoo #1: Why another podcast?How do I cover large gaps in drywall?How do I keep drywall around a patch from crumbling?Can I glue a second layer of drywall?How to patch long strip on drywall?Large drywall patch: how to avoid bulging seams?Drywall Mesh Patch vs. Bulge? To remove or not to remove?How to fix this drywall job?Prep drywall before backsplashWhat's the best way to fix this horrible drywall patch job?Drywall patching using 3M Patch Plus Primer